General

Chat

Need help with math

Don't know what I'm doing wrong here...

lim (x -> 0+) x^(1/x)

y = lim (x -> 0+) x^(1/x)

ln(y) = lim (x -> 0+) (1/x)ln(x)

ln(y) = lim (x -> 0+) ln(x)/x

As x approaches 0 from the right, the numerator approaches -∞ and the denominator approaches 0... this gives the form of (-∞/0)

Applying L'Hopital's Rule:

ln(y) = lim (x -> 0+) 1/x

As x approaches 0, the expression approaches ∞, so:

ln(y) = ∞

e^∞ = y
y = ∞

But this is wrong. What am I doing wrong?

September 22, 2012

16 Comments • Newest first

FlashedBlaze

As x approaches 0, the expression approaches ∞, so:

ln(y) = -∞

e^-∞ = y
y = -∞

That infinity should be a negative infinity. e^-infinity does equal 0.

Reply September 23, 2012 - edited
Oyster

You don't use L'Hospital's rule here, because your limit is [-Infinity/0].
[b]L'Hospital's Rule only applies for [0/0] or [Infinity/Infinity] limits. [/b]
In your case:
ln y = lim x->0+ {ln[x]}/x = {1/0}*{-Infinity} = {Infinity}*{-Infinity}={-Infinity}
ln y = {-Infinity}
....y=e^{-Infinity} = 0
....y=0

Reply September 23, 2012 - edited
DrHye

[quote=SunsetDews]Ah nice catch. You're right. 0^∞ isn't a special limit... I was taking it farther than needed

(Fairly sure e^∞ = ∞ though...)

Thanks[/quote]

Yeah it is, I was thinking of something else. But glad we figured that other part out

Reply September 22, 2012 - edited
SunsetDews

[quote=DrHye]I made an edit because I may be confusing that special case with infinity^0

I think the answer's 0. I'm just about positive 0^infinity = 0 and isn't a special case for limits

And looking at your work... e^∞ = 0 not ∞[/quote]

Ah nice catch. You're right. 0^∞ isn't a special limit... I was taking it farther than needed

(Fairly sure e^∞ = ∞ though...)

Thanks

Reply September 22, 2012 - edited
DrHye

[quote=SunsetDews]Yeah, the way I do these is usually use logarithms to bring down the exponent which will give me an indeterminate product

Then using the fact that a*b = a/(1/b), I just rewrite it to make the product a quotient and use l'hopital's rule... but in this case, the exponent goes directly to a quotient and I don't see how else I'd manipulate it. I tried rearranging the terms to make it x^(-1)/(1/ln(x)) and that didn't work out either..[/quote]

I made an edit because I may be confusing that special case with infinity^0

I think the answer's 0. I'm just about positive 0^infinity = 0 and isn't a special case for limits

Reply September 22, 2012 - edited
SunsetDews

Yeah guys I know how to use wolframalpha... the problem is I'll eventually need to show an actual procedure during an exam lol.

Reply September 22, 2012 - edited
Schokoshake

Well, just doing the old fashioned noob way of plugging in values to the right of 0, you find that the limit approaches 0 as x approaches 0 from the right.

Reply September 22, 2012 - edited
SunsetDews

[quote=DrHye]When you take the limit of the initial problem lim (x -> 0+) x^(1/x), it's 0^(infinity)
I believe this is one of the special cases of limits, and I remember there were some steps you had to take if you were given a form like the one we have... But I don't remember what to do[/quote]

Yeah, the way I do these is usually use logarithms to bring down the exponent which will give me an indeterminate product

Then using the fact that a*b = a/(1/b), I just rewrite it to make the product a quotient and use l'hopital's rule... but in this case, the exponent goes directly to a quotient and I don't see how else I'd manipulate it. I tried rearranging the terms to make it x^(-1)/(1/ln(x)) and that didn't work out either..

Reply September 22, 2012 - edited
AmyrIin

it's equal to 0!

Reply September 22, 2012 - edited
DrHye

[quote=SunsetDews]Yeah that's what's striking me as the possible error... but I don't know how else I'd manipulate the limit and I can't really interpret ∞/0.[/quote]

When you take the limit of the initial problem lim (x -> 0+) x^(1/x), it's 0^(infinity)
I believe this is one of the special cases of limits, and I remember there were some steps you had to take if you were given a form like the one we have... But I don't remember what to do. I could be wrong though

But technically, 0 to any nonzero power is 0 (if the power is 0 I believe it's undefined). Do you have the answer key?

Reply September 22, 2012 - edited
SunsetDews

[quote=DrHye]Are you allowed to use l'hopital's rule if it's -∞/0? I think it has to be 0/0 or ∞/∞[/quote]

Yeah that's what's striking me as the possible error... but I don't know how else I'd manipulate the limit and I can't really interpret ∞/0.

Reply September 22, 2012 - edited
radkai

I don't learn this before.

Reply September 22, 2012 - edited
DrHye

Are you allowed to use l'hopital's rule if it's -∞/0? I think it has to be 0/0 or ∞/∞

Reply September 22, 2012 - edited
SunsetDews

[quote=angtena]lim answers can be numbers or +-∞[/quote]

Yeah... my answer was infinity, but it's wrong. The limit's supposed to be 0.

Reply September 22, 2012 - edited
angtena

lim answers can be numbers or +-∞

Reply September 22, 2012 - edited
fun2killu

[quote=Mhmmmm]Stopped reading after "lim"[/quote]

why even post if u arn't going to help the TS? Stop spamming.

@TS: let me find my old math notes. I might be able to help you

Reply September 22, 2012 - edited